Page 1 of 1

question on assumtions and strongly supported

Posted: Sun Dec 03, 2017 10:07 am
by fire_fried_rice
Hi, I had these questions come up when I solve the question types mentioned above. For example, on strongly supported questions, there are times when the answer just jumps out. Yes, I do pick that answer but when I try to eliminate other answers, some seems to be slightly supporting. I'm not sure if this is due to me finding the 'powerful' strongly supported answer or the answer choice themselves actually being weakly supporting the argument. Preptest 10, section 1 question 9 would be an example of this. The paleiotologists think that these centipedes lived on land. D just stands out, while for me B/C and E were 'meh..I supposed you could say these do support...."

As for the assumption question, do you guys always find what the assumptions are before reading the answer choices? Sometimes I do and sometimes I don't and the fact that I do not see the assumption everytime makes me feel that I'm still incompetent at seeing the logical relationships in a stimulus. For example using preptest 10 again, on section 1 question 15, I could not see what the assumption was. The conclusion is that there is a new theory that states the it was the volcanic activity that wiped out dinosaurs and the sentence below that is its supporting detail. I was "ok..got that..but the assumption? no clue..lets read the choices to see if I can find one..." Is this a reasonable way to solve these questions? If not, I assume these questions and thoughts that I have are telling me that I still need more practice.

Re: question on assumtions and strongly supported

Posted: Sun Dec 03, 2017 10:24 am
by AJordan
Many things here you're way off with but first and foremost you need rudimentary work on recognizing question types. PT10S1Q9 is not an MSS question, it's a strengthen question. PT10S1Q15 is a necessary assumption question, not a sufficient assumption question. Though they can (and do) overlap, this is rare. Check your question stem identification knowledge. This may be where your trouble lies.

Additionally, with MSS questions you really need to get out of the habit of thinking that more than 1 answer choice will be supported. It won't. You need to pick the answer choice that is the ONLY choice supported by the information in the stimulus. The other answers are wrong. One common tip I give to my beginner students is that we're looking for an answer in an MSS question that is easy to prove, thus, be wary of "strong" answer choices on MSS questions. But again, the question you referenced was a strengthen question, not MSS.

Next, you need a primer on the difference between sufficient and necessary assumption questions. They're not the same thing. Most sufficient assumption questions, especially in the first 15 of a section, can and should be predicted as they're "gap filler" SA questions. Only the super hard ones really bring in other valid argument forms to get the job done e.g. "most"/"most"-"some" or something along those lines. You, however, referenced a necessary assumption here. Necessary assumption questions are looking for an answer that, if false, wrecks the argument. Another way to phrase that which I reference for my students when checking NA answers, I have them say, "this answer MUST BE TRUE because IF IT'S NOT this argument is WRECKED." That's a good way to check your comprehension.

Re: question on assumtions and strongly supported

Posted: Mon Dec 04, 2017 10:33 am
by Deardevil
It does not matter how weak a strengthener is as long as it does strengthen.
If I am only a millimeter taller than you, guess what? I am still taller than you...

The new theory to explain dinosaurs' extinction deals with volcanic activity. What does it assume?

(A) The massive volcanic activity was not caused by the impact of the asteroid.
Has to be because the stimulus states what about an asteroid impact? It is irrelevant.

(B) No individual dinosaurs survived the impact of the asteroid, if it occurred.
Does this HAVE to be the case? Perhaps one of them lives for an extra day or so. We have no idea!

(C) The extinctions took place over a longer time period than they would have if caused by an asteroid.
Again, must this be true?

(D) Other volcanic eruptions were not occurring at the same time as those in the Deccan regions.
What if they were? Would that destroy the argument?

(E) It is not possible to determine which would have occurred first, the volcanic flows in the Deccan region or the supposed impact of an asteroid.
So what? The stimulus is about the occurrence of one thing over the other, not the sequence.